Difference between revisions of "2002 AMC 10P Problems/Problem 5"

(Problem 5)
(Solution 2)
 
(6 intermediate revisions by the same user not shown)
Line 14: Line 14:
 
</math>
 
</math>
  
== Solution 1==
+
== Solution 1 ==
 +
The recursive rule is equal to <math>a_{n+1}=\frac{1}{3}+a_{n}</math> for all <math>n \geq 1.</math> By recursion, <math>a_{n+2}=\frac{1}{3}+a_{n+1}=a_{n+2}=\frac{1}{3}+\frac{1}{3}+a_n.</math> If we set <math>n=1</math> and repeat this process <math>2001</math> times, we will get <math>a_{2001+1}=a_{2002}=\frac{1}{3}(2001) + a_1=667+1=668.</math>
  
 +
Thus, our answer is <math>\boxed{\textbf{(C) } 668}.</math>
  
 +
== Solution 2 ==
 +
Find the first few terms of the sequence and find a pattern.
 +
<cmath>
 +
\begin{tabular}{|c|c|c|c|c|c|}
 +
\hline
 +
n & 1 & 2 & 3 & 4 & 5 \\
 +
\hline
 +
an & 1 & 4/3 & 5/3 & 2 & 7/3 \\
 +
\hline
 +
\end{tabular}
 +
</cmath>
 +
 +
From here, we can deduce that <math>a_n=\frac{2}{3}+\frac{1}{3}n.</math> Plugging in <math>n=2002,</math> <math>a_{2002}=\frac{2}{3}+\frac{2002}{3}=668.</math>
 +
 +
Thus, our answer is <math>\boxed{\textbf{(C) } 668}.</math>
  
 
== See also ==
 
== See also ==
 
{{AMC10 box|year=2002|ab=P|num-b=4|num-a=6}}
 
{{AMC10 box|year=2002|ab=P|num-b=4|num-a=6}}
 
{{MAA Notice}}
 
{{MAA Notice}}

Latest revision as of 20:55, 14 July 2024

Problem

Let $(a_n)_{n \geq 1}$ be a sequence such that $a_1 = 1$ and $3a_{n+1} - 3a_n = 1$ for all $n \geq 1.$ Find $a_{2002}.$

$\text{(A) }666 \qquad \text{(B) }667 \qquad \text{(C) }668 \qquad \text{(D) }669 \qquad \text{(E) }670$

Solution 1

The recursive rule is equal to $a_{n+1}=\frac{1}{3}+a_{n}$ for all $n \geq 1.$ By recursion, $a_{n+2}=\frac{1}{3}+a_{n+1}=a_{n+2}=\frac{1}{3}+\frac{1}{3}+a_n.$ If we set $n=1$ and repeat this process $2001$ times, we will get $a_{2001+1}=a_{2002}=\frac{1}{3}(2001) + a_1=667+1=668.$

Thus, our answer is $\boxed{\textbf{(C) } 668}.$

Solution 2

Find the first few terms of the sequence and find a pattern. \[\begin{tabular}{|c|c|c|c|c|c|}  \hline   n & 1 & 2 & 3 & 4 & 5 \\   \hline  an & 1 & 4/3 & 5/3 & 2 & 7/3 \\  \hline \end{tabular}\]

From here, we can deduce that $a_n=\frac{2}{3}+\frac{1}{3}n.$ Plugging in $n=2002,$ $a_{2002}=\frac{2}{3}+\frac{2002}{3}=668.$

Thus, our answer is $\boxed{\textbf{(C) } 668}.$

See also

2002 AMC 10P (ProblemsAnswer KeyResources)
Preceded by
Problem 4
Followed by
Problem 6
1 2 3 4 5 6 7 8 9 10 11 12 13 14 15 16 17 18 19 20 21 22 23 24 25
All AMC 10 Problems and Solutions

The problems on this page are copyrighted by the Mathematical Association of America's American Mathematics Competitions. AMC logo.png